Difference between revisions of "2006 IMO Problems/Problem 5"
m (→Hint) |
m (→Solution 2) |
||
(3 intermediate revisions by the same user not shown) | |||
Line 55: | Line 55: | ||
Construct a polynomial to give the final contradiction by keeping in mind the theorem: | Construct a polynomial to give the final contradiction by keeping in mind the theorem: | ||
If the equation f(x)=<math>a</math> (<math>a</math> is a constant)has more roots than deg(f), then f=a. | If the equation f(x)=<math>a</math> (<math>a</math> is a constant)has more roots than deg(f), then f=a. | ||
+ | |||
+ | == Solution 2 == | ||
+ | |||
+ | Note: | ||
+ | We will denote <math> P(...(P(x))....) </math> (where <math>P</math> occurs <math>k</math> times) as <math>P_k(x)</math> throughout the proof, for any integer <math>k>1</math>. | ||
+ | |||
+ | For the sake of contradiction, let the problem statement be false. | ||
+ | For convinience's sake, let the stated equation have <math>n+1</math> solutions: | ||
+ | <math> P_k(x_1)</math> = <math>x_1</math>,...,<math>P_k(x_{n+1})</math> = <math>x_{n+1}</math>. | ||
+ | |||
+ | Without loss of generality, suppose <math>x_1 < ... <x_{n+1}</math>. | ||
+ | |||
+ | As P is defined over the integers, <math>x_i - x_j \mid P(x_i) - P(x_j)</math>. | ||
+ | Also, <math>P(x_i) - P(x_j) \mid P_2(x_i) - P_2(x_j) \mid ... \mid P_k(x_i) - P_k(x_j) = x_i - x_j.</math> | ||
+ | Hence, <math>|P(x_i) - P(x_j)| = x_i - x_j</math> for all <math>i<j</math>. | ||
+ | |||
+ | Claim. | ||
+ | |||
+ | <math>P(x_i) \neq P(x_j)</math> for any <math>i \neq j</math>. | ||
+ | |||
+ | Proof. | ||
+ | |||
+ | For the sake of contradiction, suppose the claim statement is wrong. | ||
+ | Without loss of generality, suppose <math>i>j</math>. | ||
+ | |||
+ | Then, <math>P(x_i)=P(x_j) => P(x_i)-P(x_j)=0 => |P(x_i)-P(x_j)|=0 => x_i - x_j = 0 => x_i=x_j</math>, a contradiction. | ||
+ | |||
+ | Hence, our claim is proved. | ||
+ | |||
+ | |||
+ | Claim. | ||
+ | |||
+ | <math>P(x_1),...,P(x_{n+1})</math> is either increasing or decreasing. | ||
+ | |||
+ | Proof. | ||
+ | |||
+ | We will only prove the increasing part, as the decreasing part is similar. | ||
+ | |||
+ | For the sake of contradiction, suppose the claim statement is wrong. | ||
+ | Then, <math>P(x_{i-1})>P(x_i)</math> and <math>P(x_i)<P(x_{i+1})</math> for some integer <math>i>1</math>. | ||
+ | |||
+ | For convinience's sake, suppose <math>i=2</math>. | ||
+ | |||
+ | Then, <math>|P(x_1)-P(x_2)|=P(x_1)-P(x_2)=x_2-x_1</math> and <math>|P(x_3)-P(x_2)|=P(x_3)-P(x_2)=x_3-x_2</math>. | ||
+ | |||
+ | Adding, we get that <math>(P(x_1)+P(x_3))-2 \cdot P(x_2) = x_3 - x_1</math>. | ||
+ | |||
+ | Now, <math>x_3 - x_1</math> will either be substituted by <math>P(x_1)-P(x_3)</math> or <math>P(x_3)-P(x_1)</math>. | ||
+ | |||
+ | Either way, we get a contradiction to our previous claim. | ||
+ | |||
+ | Hence, <math>P(x_2)>P(x_3)</math>. | ||
+ | Similarly, <math>P(x_3)>P(x_4)</math>,...,<math>P(x_n)>P(x_{n+1})</math>. | ||
+ | |||
+ | Then, <math>P(x_1)>...>P(x_{n+1})</math>. | ||
+ | |||
+ | One may similarly show the decreasing part. | ||
+ | |||
+ | Hence, our claim is proved. | ||
+ | |||
+ | |||
+ | Case 1, The sequence <math>P(x_1),...,P(x_{n+1})</math> is decreasing: | ||
+ | |||
+ | Let <math>T(x)=(P(x)-x)-(P(x_1)-x_1)</math>. | ||
+ | |||
+ | Then, <math>T(x)</math> has <math>n+1</math> roots: <math>x_1,...,x_{n+1}</math>, while <math>deg(T)=deg(P)=n</math>, a contradiction. | ||
+ | |||
+ | One may similarly show the contradiction if the sequence <math>P(x_1),...,P(x_{n+1})</math> is increasing. | ||
+ | |||
+ | |||
+ | Hence, we get our desired contradiction, and thus we are done. | ||
== Resources == | == Resources == |
Latest revision as of 05:41, 1 October 2025
Contents
Problem
(Dan Schwarz, Romania)
Let be a polynomial of degree
with integer coefficients, and let
be a positive integer. Consider the polynomial
, where
occurs
times. Prove that there are at most
integers
such that
.
Solution
We use the notation for
.
Lemma 1. The problem statement holds for .
Proof. Suppose that ,
are integers such that
and
for all indices
. Let the set
have
distinct elements. It suffices to show that
.
If for all indices
, then the polynomial
has at least
roots; since
is not linear, it follows that
by the division algorithm.
Suppose on the other hand that , for some index
. In this case, we claim that
is constant for every index
. Indeed, we note that
so
. Similarly,
so
. It follows that
.
This proves our claim. It follows that the polynomial
has at least
roots. Since
is not linear it follows again that
, as desired. Thus the lemma is proven.
Lemma 2. If is a positive integer such that
for some positive integer
, then
.
Proof. Let us denote , and
, for positive integers
. Then
, and
It follows that
is constant for all indices
; let us abbreviate this quantity
. Now, since
it follows that for some index
,
or
. Since
, it then follows that
, as desired.
Now, if there are more than integers
for which
, then by Lemma 2, there are more than
integers
such that
, which is a contradiction by Lemma 1. Thus the problem is solved.
Alternate solutions are always welcome. If you have a different, elegant solution to this problem, please add it to this page.
Hints
Try to use contradiction:
Q()=
,
,Q(
)=
. WLOG suppose
.
Think of using the theorem .
You will slowly get that |P()-P(
)|=
(i<j).
Try to prove that P(),...,P(
) is either increasing or decreasing.
Construct a polynomial to give the final contradiction by keeping in mind the theorem:
If the equation f(x)= (
is a constant)has more roots than deg(f), then f=a.
Solution 2
Note:
We will denote (where
occurs
times) as
throughout the proof, for any integer
.
For the sake of contradiction, let the problem statement be false.
For convinience's sake, let the stated equation have solutions:
=
,...,
=
.
Without loss of generality, suppose .
As P is defined over the integers, .
Also,
Hence,
for all
.
Claim.
for any
.
Proof.
For the sake of contradiction, suppose the claim statement is wrong.
Without loss of generality, suppose .
Then, , a contradiction.
Hence, our claim is proved.
Claim.
is either increasing or decreasing.
Proof.
We will only prove the increasing part, as the decreasing part is similar.
For the sake of contradiction, suppose the claim statement is wrong.
Then, and
for some integer
.
For convinience's sake, suppose .
Then, and
.
Adding, we get that .
Now, will either be substituted by
or
.
Either way, we get a contradiction to our previous claim.
Hence, .
Similarly,
,...,
.
Then, .
One may similarly show the decreasing part.
Hence, our claim is proved.
Case 1, The sequence is decreasing:
Let .
Then, has
roots:
, while
, a contradiction.
One may similarly show the contradiction if the sequence is increasing.
Hence, we get our desired contradiction, and thus we are done.
Resources
- 1974 USAMO Problems/Problem 1, which implies a special case of this problem
2006 IMO (Problems) • Resources | ||
Preceded by Problem 4 |
1 • 2 • 3 • 4 • 5 • 6 | Followed by Problem 6 |
All IMO Problems and Solutions |